Stochastic matrix problem

The name of the pictureThe name of the pictureThe name of the pictureClash Royale CLAN TAG#URR8PPP











up vote
0
down vote

favorite













A (left) stochastic matrix is one which has only non-negative entries, and such that the entries in each column sums up to $1$. Let $A$ be any (general) $2 times 2$ stochastic matrix.



a) Show that one of the eigenvalues of $A$ is $1$.



b) Show that a general $n times n$ stochastic matrix also has one eigenvalue equal to $1$.



c) What happens if the column sums are $1$ as above, but the entries may be any (possibly negative) real numbers?




I'm not sure how to start with this question. Should I explain using the Perron-Frobenius theorem? How to show for (a)? Because I think I can use the theorem only for (b), or is there a simpler explanation? And I'm not sure how to answer (c) as well.







share|cite|improve this question


























    up vote
    0
    down vote

    favorite













    A (left) stochastic matrix is one which has only non-negative entries, and such that the entries in each column sums up to $1$. Let $A$ be any (general) $2 times 2$ stochastic matrix.



    a) Show that one of the eigenvalues of $A$ is $1$.



    b) Show that a general $n times n$ stochastic matrix also has one eigenvalue equal to $1$.



    c) What happens if the column sums are $1$ as above, but the entries may be any (possibly negative) real numbers?




    I'm not sure how to start with this question. Should I explain using the Perron-Frobenius theorem? How to show for (a)? Because I think I can use the theorem only for (b), or is there a simpler explanation? And I'm not sure how to answer (c) as well.







    share|cite|improve this question
























      up vote
      0
      down vote

      favorite









      up vote
      0
      down vote

      favorite












      A (left) stochastic matrix is one which has only non-negative entries, and such that the entries in each column sums up to $1$. Let $A$ be any (general) $2 times 2$ stochastic matrix.



      a) Show that one of the eigenvalues of $A$ is $1$.



      b) Show that a general $n times n$ stochastic matrix also has one eigenvalue equal to $1$.



      c) What happens if the column sums are $1$ as above, but the entries may be any (possibly negative) real numbers?




      I'm not sure how to start with this question. Should I explain using the Perron-Frobenius theorem? How to show for (a)? Because I think I can use the theorem only for (b), or is there a simpler explanation? And I'm not sure how to answer (c) as well.







      share|cite|improve this question















      A (left) stochastic matrix is one which has only non-negative entries, and such that the entries in each column sums up to $1$. Let $A$ be any (general) $2 times 2$ stochastic matrix.



      a) Show that one of the eigenvalues of $A$ is $1$.



      b) Show that a general $n times n$ stochastic matrix also has one eigenvalue equal to $1$.



      c) What happens if the column sums are $1$ as above, but the entries may be any (possibly negative) real numbers?




      I'm not sure how to start with this question. Should I explain using the Perron-Frobenius theorem? How to show for (a)? Because I think I can use the theorem only for (b), or is there a simpler explanation? And I'm not sure how to answer (c) as well.









      share|cite|improve this question













      share|cite|improve this question




      share|cite|improve this question








      edited Aug 17 at 7:37









      Rodrigo de Azevedo

      12.6k41751




      12.6k41751










      asked Feb 20 '14 at 23:17









      user115636

      8119




      8119




















          1 Answer
          1






          active

          oldest

          votes

















          up vote
          1
          down vote













          Hint: a matrix $A$ has eigenvalue $1$ if and only if $det(A-I)=0$. Now look at $A-I$ and see what you get if you evaluate the determinant by adding the rows.






          share|cite|improve this answer




















            Your Answer




            StackExchange.ifUsing("editor", function ()
            return StackExchange.using("mathjaxEditing", function ()
            StackExchange.MarkdownEditor.creationCallbacks.add(function (editor, postfix)
            StackExchange.mathjaxEditing.prepareWmdForMathJax(editor, postfix, [["$", "$"], ["\\(","\\)"]]);
            );
            );
            , "mathjax-editing");

            StackExchange.ready(function()
            var channelOptions =
            tags: "".split(" "),
            id: "69"
            ;
            initTagRenderer("".split(" "), "".split(" "), channelOptions);

            StackExchange.using("externalEditor", function()
            // Have to fire editor after snippets, if snippets enabled
            if (StackExchange.settings.snippets.snippetsEnabled)
            StackExchange.using("snippets", function()
            createEditor();
            );

            else
            createEditor();

            );

            function createEditor()
            StackExchange.prepareEditor(
            heartbeatType: 'answer',
            convertImagesToLinks: true,
            noModals: false,
            showLowRepImageUploadWarning: true,
            reputationToPostImages: 10,
            bindNavPrevention: true,
            postfix: "",
            noCode: true, onDemand: true,
            discardSelector: ".discard-answer"
            ,immediatelyShowMarkdownHelp:true
            );



            );








             

            draft saved


            draft discarded


















            StackExchange.ready(
            function ()
            StackExchange.openid.initPostLogin('.new-post-login', 'https%3a%2f%2fmath.stackexchange.com%2fquestions%2f684162%2fstochastic-matrix-problem%23new-answer', 'question_page');

            );

            Post as a guest






























            1 Answer
            1






            active

            oldest

            votes








            1 Answer
            1






            active

            oldest

            votes









            active

            oldest

            votes






            active

            oldest

            votes








            up vote
            1
            down vote













            Hint: a matrix $A$ has eigenvalue $1$ if and only if $det(A-I)=0$. Now look at $A-I$ and see what you get if you evaluate the determinant by adding the rows.






            share|cite|improve this answer
























              up vote
              1
              down vote













              Hint: a matrix $A$ has eigenvalue $1$ if and only if $det(A-I)=0$. Now look at $A-I$ and see what you get if you evaluate the determinant by adding the rows.






              share|cite|improve this answer






















                up vote
                1
                down vote










                up vote
                1
                down vote









                Hint: a matrix $A$ has eigenvalue $1$ if and only if $det(A-I)=0$. Now look at $A-I$ and see what you get if you evaluate the determinant by adding the rows.






                share|cite|improve this answer












                Hint: a matrix $A$ has eigenvalue $1$ if and only if $det(A-I)=0$. Now look at $A-I$ and see what you get if you evaluate the determinant by adding the rows.







                share|cite|improve this answer












                share|cite|improve this answer



                share|cite|improve this answer










                answered Feb 20 '14 at 23:21









                David

                66.1k662125




                66.1k662125






















                     

                    draft saved


                    draft discarded


























                     


                    draft saved


                    draft discarded














                    StackExchange.ready(
                    function ()
                    StackExchange.openid.initPostLogin('.new-post-login', 'https%3a%2f%2fmath.stackexchange.com%2fquestions%2f684162%2fstochastic-matrix-problem%23new-answer', 'question_page');

                    );

                    Post as a guest













































































                    這個網誌中的熱門文章

                    Is there any way to eliminate the singular point to solve this integral by hand or by approximations?

                    Why am i infinitely getting the same tweet with the Twitter Search API?

                    Carbon dioxide